Linear Approximation of Angles

Click For Summary
The discussion focuses on calculating the change in distance of a basketball shot based on a change in launch angle using linear approximation. The formula used is s = (1/32)v² * sin(2Θ), where the derivative is calculated to find the change in distance, resulting in approximately 0.255 ft for a small angle change. The participant concludes that a 2-degree deviation in angle would likely result in a missed shot, as the calculated change in distance would lead to a total distance of 19.23 ft, exceeding the necessary 18.1 ft. Clarifications on notation and calculations were provided, confirming the approach was correct. The discussion emphasizes the importance of precise angle measurement in successful basketball shooting.
Onodeyja
Messages
9
Reaction score
0

Homework Statement



A player located 18.1 ft from a basket launches a successful jump shot from a height of 10 ft (level with the rim of the basket), at an angle Θ = 34 degrees and initial velocity of v = 25 ft/s.A. Show that the distance s of the shot changes by approximately 0.255∆Θ ft if the angle changes by an amount ∆Θ. Remember to convert the angle to radians in the Linear Approximation.

B. Is it likely that the shot would have been successful if the angle were off by 2 degrees?

Homework Equations



s = (1/32)v² * sin(2Θ)

The Attempt at a Solution



I'm not sure if I'm going in the right direction, but here's what I have.

∆f = f'(a)∆Θ

f' = 1250/32 * cos(2Θ)

∆f = f'(34 * (∏/180)) * ∆Θ
∆f = 1250/32 * cos(2 * (17∏/90)) * ∆Θ
∆f = 1250/32 * cos (17∏/90) * ∆Θ
∆f = 32.38∆Θ = 0.255
∆Θ = 0.255/32.38 = 0.00787 * (180/∏) = 0.45˚

Part B: No, the shot wouldn't be successful.
 
Last edited:
Physics news on Phys.org
Your statement
∆f = f'(a)∆Θ
Certainly seems to be correct, modulo notation. In particular, you meant f=s, a =\theta right?

After that, your calculations looks fine.

Also, what is your motivation for answering "No" to part b? Can you numerically justify it?
 
Kreizhn said:
Your statement
∆f = f'(a)∆Θ
Certainly seems to be correct, modulo notation. In particular, you meant f=s, a =\theta right?

After that, your calculations looks fine.

Also, what is your motivation for answering "No" to part b? Can you numerically justify it?

Yes, I should have written the function as s instead of f.

Θ = 34˚ = 17∏/90
∆Θ = 2˚ = ∏/90

∆s = s'(Θ)∆Θ = s'(17∏/90)(∏/90)
∆s = [1250/32 * cos(17∏/90)] * (∏/90)
∆s = 1.13

18.1 ft + 1.13 ft = 19.23 ft
 
Last edited:
Question: A clock's minute hand has length 4 and its hour hand has length 3. What is the distance between the tips at the moment when it is increasing most rapidly?(Putnam Exam Question) Answer: Making assumption that both the hands moves at constant angular velocities, the answer is ## \sqrt{7} .## But don't you think this assumption is somewhat doubtful and wrong?

Similar threads

Replies
5
Views
2K
  • · Replies 2 ·
Replies
2
Views
2K
Replies
8
Views
5K
  • · Replies 2 ·
Replies
2
Views
2K
  • · Replies 4 ·
Replies
4
Views
2K
Replies
2
Views
4K
  • · Replies 8 ·
Replies
8
Views
3K
Replies
2
Views
3K
  • · Replies 2 ·
Replies
2
Views
10K
  • · Replies 18 ·
Replies
18
Views
6K